Find the perimeter of quadrilateral ABCD with vertices A(0, 4), B(4, 1), C(1, -3), and D(-3, 0).

Answers

Answer 1

Given:

The vertices of a quadrilateral ABCD are A(0, 4), B(4, 1), C(1, -3), and D(-3, 0).

To find:

The perimeter of quadrilateral ABCD.

Solution:

Distance formula:

[tex]d=\sqrt{(x_2-x_1)^2+(y_2-y_1)^2}[/tex]

Using the distance formula, we get

[tex]AB=\sqrt{(4-0)^2+(1-4)^2}[/tex]

[tex]AB=\sqrt{(4)^2+(-3)^2}[/tex]

[tex]AB=\sqrt{16+9}[/tex]

[tex]AB=\sqrt{25}[/tex]

[tex]AB=5[/tex]

Similarly,

[tex]BC=\sqrt{(1-4)^2+(-3-1)^2}[/tex]

[tex]BC=5[/tex]

[tex]CD=\sqrt{(-3-1)^2+(0-(-3))^2}[/tex]

[tex]CD=5[/tex]

And,

[tex]AD=\sqrt{(-3-0)^2+(0-4)^2}[/tex]

[tex]AD=5[/tex]

Now, the perimeter of the quadrilateral ABCD is:

[tex]P=AB+BC+CD+AD[/tex]

[tex]P=5+5+5+5[/tex]

[tex]P=20[/tex]

Therefore, the perimeter of the quadrilateral ABCD is 20 units.


Related Questions

The formula for the circumference of a circle is R = c/2(pi)

Find the radius of a circle that has a circumference of 16(pi)

A) r = 4
B) r = 8
C) r = 12
D) r = 16

Answers

The answer is B) r= 8

Evaluate 2y when y = 6y

Answers

I believe the answer is 12y
im sure the answer is 12y

0.2(x + 20) – 3 > –7 – 6.2x

Answers

Answer:

x > - 1.25

Step-by-step explanation:

0.2(x + 20) – 3 > –7 – 6.2x

0.2x + 4 - 3 > - 7 - 6.2x

0.2x + 1 > - 7 - 6.2x

Collect like terms

0.2x + 6.2x > -7 - 1

6.4x > -8

x > - 8/6.4

x > - 1.25

Note:

The > didn't change because you didn't divide by a negative value

Inequality signs changes when divided by a negative value

Answer:

Step-by-step explanation:

-1.25

Select the expression that represents the following statement: add 24 to the quotient of 16 and 8.

Answers

Answer:

16/8 + 24

Step-by-step explanation:

The area of the rectangular sandbox at Dave's school is 117 square feet. The sandbox has a width of 9 feet as shown in the diagram. What is the perimeter of the sandbox?

Answers

Answer:

ay bru ima tell yu dhis rn is c

Step-by-step explanation:

Hello, Brainly community!

This question is for all of those Calculus people out there.

The volume of a swimming pool is changing with respect to time, such that the volume is given by W(t), where W(t) is measured in cubic centimeters and t is measured in seconds. A tangent line is shown for W(t) at t = 3 seconds. Determine the best estimate for the value of the instantaneous rate of change of W(t) when t = 3.
(I've narrowed down the answer choices to 2, and just really need to find the right way of thinking to find the answer)

(A) W(lim t) as t goes to 3.
(B) [W(3.1) - W(2.9)] / 0.2.

Thank you in advance!

Answers

Answer:

(B)  [tex]\displaystyle \frac{W(3.1) - W(2.9)}{0.2}[/tex]

General Formulas and Concepts:

Calculus

Limits

Derivatives

The definition of a derivative is the slope of the tangent line.

Derivative Notation

Instantaneous Rates

Tangent Line: [tex]\displaystyle f'(x) = \frac{f(b) - f(a)}{b - a}[/tex]

Step-by-step explanation:

Since we are trying to find a rate at which W(t) changes, we must find the derivative at t = 3.

We are given 2 close answer choices that would have the same numerical answer but different meanings:

(A)  [tex]\displaystyle \lim_{t \to 3} W(t)[/tex](B)  [tex]\displaystyle \frac{W(3.1) - W(2.9)}{0.2}[/tex]

If we look at answer choice (A), we see that our units would simply just be volume. It would not have the units of a rate of change. Yes, it may be the closest numerically correct answer, but it does not tell us the rate at which the volume would be changing and it is not a derivative.

If we look at answer choice (B), we see that our units would be cm³/s, and that is most certainly a rate of change. Answer choice (B) is also a derivative at t = 3, and a derivative tells us what rate something is changing.

∴ Answer choice (B) will give us the best estimate for the value of the instantaneous rate of change of W(t) when t = 3.

Topic: AP Calculus AB/BC (Calculus I/I + II)

Unit: Differentiation

Book: College Calculus 10e

If a bus travel for 120 minutes at a speed of 75 kilometers per hour how far has the bus traveled?

Answers

Answer:

150 km

Step-by-step explanation:

Put the minutes into hours 120min is 2 hours.

Distance = speed * time

Distance = 75 * 2

Distance = 150

Answer:

150 kilometers

Step-by-step explanation:

if the bus is going 75 kilometers an hour and they traveled for 120 minutes (exactly 2 hours) then you would just multiply 75 by 2 to get 150 kilometers total.

There are 35 times as many students at Wow University as teachers. When all the students and
teachers are seated in the 8544 seat auditorium, 12 seats are empty. How many students attend
Wow University?

Answers

Given:

There are 35 times as many students at Wow University as teachers.

When all the students and teachers are seated in the 8544 seat auditorium, 12 seats are empty.

To find:

The total number of students.

Solution:

Let x be the number of teachers at Wow University. So, the number of student is :

[tex]35\times x=35x[/tex]

When all the students and teachers are seated in the 8544 seat auditorium, 12 seats are empty.

[tex]x+35x=8544-12[/tex]

[tex]36x=8532[/tex]

[tex]x=\dfrac{8532}{36}[/tex]

[tex]x=237[/tex]

The number of total students is:

[tex]35x=35(237)[/tex]

[tex]35x=8295[/tex]

Therefore, the total number of students is 8295.

Find ∠MPN

Help me please

Answers

Answer:

[tex]22^{\circ}[/tex]

Step-by-step explanation:

Line [tex]\overline{PM}[/tex] is a diameter of the circle because it passes through the circle's center O. Therefore, arc [tex]\widehat{PLM}[/tex] must be 180 degrees, as these are 360 degree in a circle.

We can then find the measure of arc [tex]\widehat{LM}[/tex]:

[tex]\widehat{LP}+\widehat{LM}=180^{\circ},\\92^{\circ}+\widehat{LM}=180^{\circ},\\\widehat{LM}=88^{\circ}[/tex]

Arc [tex]\widehat{LM}[/tex] is formed by angle [tex]\angle LPM[/tex]. Define an inscribed angle by an angle with a point on the circle creating an arc on the circumference of the circle. The measure of an inscribed angle is exactly half of the measure of the arc it forms.

Therefore, the measure of [tex]\angle LPM[/tex] must be:

[tex]m\angle LPM=\frac{88}{2}=44^{\circ}[/tex]

Similarly, the measure of [tex]\angle LNP[/tex] must be:

[tex]m\angle LNP=\frac{92}{2}=46^{\circ}[/tex]

Angles [tex]\angle LPM[/tex] and [tex]\angle MPN[/tex] form angle [tex]\angle LPN[/tex], which is one of the three angles in [tex]\triangle LPN[/tex]. Since the sum of the interior angles of a triangle add up to 180 degrees, we have:

[tex](\angle MPN+\angle LPM)+\angl+ PLN+\angle LNP=180^{\circ},\\\angle MPN+44+46+68=180,\\\angle MPN=180-44-46-68,\\\angle MPN=\boxed{22^{\circ}}[/tex]

Find value of x.
A. 110
B. 47
C. 68
D. 112

Answers

Answer:

B

Step-by-step explanation:

The sum of the inner angles of a quadrilateral is 360 degrees

135 + 110 + 68 + x = 360

313 + x = 360

x = 47 degrees

Answer:

47

Step-by-step explanation:

whole thing is 360 degrees

68 + 110 + 135 = 313

360 - 313 = 47

x looks small too (if you had to guess in a multiple choice question)

Arturo is building a flower bed in the shape of a right triangle. The hypotenuse of the right triangle is 13 feet. One of the legs needs to be 7 feet longer than the other leg.

Which equation can be used to find x, the length of the shorter leg?

x(x + 7) = 13
x(x + 7) = 169
x2 + (x + 7)2 = 13
x2 + (x + 7)2 = 169

Answers

Answer:

x(x + 7) = 169

Step-by-step explanation:

We are given that

Let length of shorter leg of right triangle = x ft

Length of other leg = x+7 ft

Hypotenuse of right triangle = 13 ft

We have to find the equation which can be used to find x, the length of shorter leg.

By pythagorous theorem

(Hypotenuse)^2=(Base)^2+(Perpendicular\;side)^2

Substitute the values then, we get

(13)^2=x^2+(x+7)^2

x^2+(x+7)^2=169

This is required equation which can used to find x.

Answer:

d. x2 + (x + 7)2 = 169

Step-by-step explanation:

use Pythagorean theorem, substitute in the given values, and you get this

pls help me don't know what to do

Answers

Answer:

x=15

Step-by-step explanation:

The 60 degree angle and the (x+45) degree angle are both the same degree because they are vertical angles.

So to solve, just subtract 45 from 60

60-45=15

That's your answer!

Hope this helps!

Greg buys 60 garden plants at a cost price of $2.00 each to sell in his shop. He sells 25 of them at the profit of 75% and 18 of them at the profit of 35%. He sells the rest of the plants for 4/5 of the cost price calculate the profit or loss he makes from selling 60 plants stating if it is a profit or loss

Answers

Answer:

$43.30 profit

Step-by-step explanation:

Total cost of plant:

60*2 = 120

Greg makes total of:

25*(2 + 0.75*2) + 18*(2 + 0.35*2) + (60 - 25 - 18)*2*4/5 = 163.3

Since Greg mare than cost, he has a profit and the amount is:

163.3 - 120 = 43.3

Choose the smallest number 3 1/8 or 10/3

Answers

Answer:

10/3

Step-by-step explanation:

31/8= 3.8

10/3= 3.3

2 1/4 x 3 1/5 brainliest

Answers

Answer:

36/5

Step-by-step explanation:

9/4×16/5

144/20

36/5

hope this is helpful

Answer:

[tex]7\frac{1}{5}[/tex]

Step-by-step explanation:

1. start by turning the fractions improper fractions:

[tex]2\frac{1}{4} =\frac{9}{4}[/tex]

[tex]3\frac{1}{5} =\frac{16}{5}[/tex]

2. then multiply them together:

[tex]\frac{9}{4}[/tex] x [tex]\frac{16}{5}[/tex] = [tex]\frac{144}{20}[/tex]

3. then simplify the fraction:

[tex]\frac{144}{20}[/tex][tex]=\frac{36}{5}[/tex]

4. turn it into a proper fraction:

[tex]\frac{36}{5} =7\frac{1}{5}[/tex]

A cinema is doing a promotion to celebrate their 50th anniversary for 1 week. They give

away a free drink to every 98th customer, a free bag of popcorn to every 112th customer and

a free cinema ticket to every 224th customer. Which lucky customer will be the first to

receive all 3 items?​

Answers

Answer:

1,568 customer

Step-by-step explanation:

Find the lowest common multiple of 98, 112, and 224

98 = 98, 196, 294, 392, 490, 588, 686, 784, 882, 980, 1078, 1176, 1274, 1372, 1470, 1568, 1666

112 = 112, 224, 336, 448, 560, 672, 784, 896, 1008, 1120, 1232, 1344, 1456, 1568, 1680, 1792, 1904

224 = 224, 448, 672, 896, 1120, 1344, 1568, 1792, 2016, 2240

The lowest common multiple of 98, 112, and 224 is 1568

Therefore, the 1,568th customer will be the first to receive all 3 iitem

The center of the circle is at the point
, and its radius is
units. The equation of this circle in standard form is
.

Answers

Is there a picture that goes with this?

D=22/7×d-90 Solve the equation
Find D​
Fast!

Answers

Answer:

D=22-90+22d/7

Step-by-step explanation:

D=22/7×d-90

D=-90+22d/7

Answer is- d=-630/-22x+7

What is the quotient of (x^3 - 3x^2 + 3x - 2) ÷ (x^2 - x + 1)?
O x - 2
O x + 2
O x- 4
O x + 1

Answers

Answer:

x-2

The choose (1)

Step-by-step explanation:

(x³-3x²+3x-2)÷(x²-x+1)

(x-2)(x²-x+1) ÷ (x²-x+1)

Delete (x²-x+1)

so = (x-2)

On the unit circle, which of the following angles has the terminal point
coordinates.
A. 45
B. 135
C. 225
D. 315

Answers

Answer: C. 225

Step-by-step explanation:

jordan wants to save to buy a car and decides to open a banking account that is offering an interest rate of 4.5% compounded annually how much will jordan have in the account after 5 years it he deposits $7,000 today?

Answers

Answer:

8,723.27$

Step-by-step explanation:

Which operation will solve the following word problem? Jeff earns $14.00 per hour, Tom earns half as much as Jeff. How much does Tom earn per hour?


Multiplication


Subtraction


Addition


Division

Answers

Answer:

The correct option is (d).

Step-by-step explanation:

Given that,

Jeff earns $14.00 per hour.

Tom earns half as much as Jeff.

We need to find the amount earn by Tom per hour.

Tom's amount = Jeff's amount/2

So,

[tex]T=\dfrac{14}{2}\\\\T=\$7[/tex]

So, Tom earn $7 per hour. Hence, division operation is used. Jeff's amount is divided by 2.

According to the number line, what is the distance between points A and B?

0 6 units
7 units
O 12 units
O 14 units

Answers

Answer:

14 units

Step-by-step explanation:

A = - 2, B = 12

Therefore,

d(A, B) = 12 - (-2) = 12 + 2 = 14 units

What’s the equation of the line that passes through the point (-4,4) and has a slope of 3/4

Answers

Answer:

y-y1=m(x-x1)

y-4=3/4(x+4)

y=3/4x+7

lidentify the domain of the function shown in the graph
O A 15257
O B. 19334
O C. 221
O D. All real numbers

Answers

Answer:

B.

Step-by-step explanation:

the visible line is the defined function.

this line goes from x=1 to x=4, and has the functional results from y=1 to y=7.

the domain is the valid interval of the input variable (typically x), while the range is the valid inescapable of the result variable (typically y).

so, B is the right answer.

Can someone help me with this math homework please!

Answers

Answer:

1: the number of years since 2008 2. t is greater than or equal to 0 3. negative values 4. continuous

Step-by-step explanation:

1. t usually represents time

2. t must be greater than 0 as you can not go backward in time

3. the range must be positive as you can not have negative bobcats

4. its continuous because its a quadratic equation

1. no. of bobcats since 2008

2. greater than equal to 0

3 negative values

4. discrete because no. of bobcats cannot be broken into fraction.

find the domain of f(x)=sec(2x)

Answers

Answer:

*Refer the image attached

Step-by-step explanation:

*Refer the image attached

What is the scale of the y-axis in this coordinate graph?


A. 1 tick mark represents 1 unit
B. 1 tick mark represents 8 units
C. 1 tick mark represents 12 units
D. 1 tick mark represents 16 units

Answers

Answer:

Obviously B

Step-by-step explanation:

A statistics professor asked students in a class their ages. On the basis of this information, the professor states that the average age of all the students in the university is 24 years. This is an example of

Answers

Answer:

propbability ???

Step-bp explanation:

Answer:

Step-by-step explanation:

This is an example of a statistical mean.

PLS HELP SOON WILL MARK BRAINLYEST

A railroad tunnel is shaped like a semi-ellipse, as shown below. A semiellipse is shown on the coordinate plane with vertices on the x axis and one point of intersection with the positive y axis. The height of the tunnel at the center is 35 ft, and the vertical clearance must be 21 ft at a point 8 ft from the center. Find an equation for the ellipse.

Answers

According to the question

b= 35 and (8,21) lies on the ellipse

After calculation we get a= 10

equation for the ellipse.

[tex] \frac{ {x}^{2} }{100} + \frac{ {y}^{2} }{1225} = 1[/tex]

Other Questions
A city has a population of 350,000 peopleSuppose that each year the population grows by 7.75%What will the population be after 6 years Use the calculator provided and round your answer to the nearest whole number If f(x) = k where k is a constant, and the points (6,1) and (8,1) lie on the graph of y = f(x), what is the value of f(0)?I think the answer is 1 but not 100% sure. I really need help with this problem you are making meat loaf with yield: 50, 4oz portions what is the total recipe cost What is the product of 3/5 and 25. Is the product more or less than 14? Explain your answer in complete sentences.(3/5 is a fraction Not Disvison ) Please help me here. I've tried this one over and over and I'm unable to get the answer that was given. How do I find the area of this triangle? h e l p :')Which of the following was not in the upper room praying before Pentecost?JamesJudeJohnJudas 15. ABCD is a cyclic quadrilateral in whichAB = BC and ABC = 70.AD produced meets BC produced at thepoint P, where APB = 30.Calculatea) ADBb) ABD Joshua is a citizen of the United States. He practices the religion of Judaism freely. Which amendment to the Constitution gives Joshua this freedom? Let u = . Find 4u. help......................... To leave the gravitational pull of the Earth, and explore other planets, satellites must have at least: find the sum of the series 2 - 2 + 22 +__+642. PLEASE HELP THIS QUESTION IS SO ANNOYING!! Please help asap, how do I find the missing variables? (Trigonometry) Evaluate 4 - x, when x = -6 Based on the graph, find the set of all x-values for which the points P(x,y) are on the graph y>0. Enter your answer using interval notation 6. By default,how the table headings are placed. (2 Points) bold and italic bold and centered bold and underlined bold and aligned Why is it difficult to establish a society on an island? If the price of a gallon of gas was $0.89 in 1984 and was $3.59 in 2014, what was the average rate of change in the price per gallon of gas?